LSAT and Law School Admissions Forum

Get expert LSAT preparation and law school admissions advice from PowerScore Test Preparation.

User avatar
 Dave Killoran
PowerScore Staff
  • PowerScore Staff
  • Posts: 5850
  • Joined: Mar 25, 2011
|
#27088
Complete Question Explanation
(The complete setup for this game can be found here: lsat/viewtopic.php?t=11359)

The correct answer choice is (E)

From the setup discussion, we know that W is always on the second or third shelf. Hence, answer choice (E) is correct.
 sotor26
  • Posts: 13
  • Joined: Oct 29, 2014
|
#17212
Hello,

I am having a little trouble understanding why E is correct. Although I know that Woolf is not on the first shelf, I don't understand why A can not be true. A linguistic monograph has to be in the first shelf in all situations, correct (P or S).
 Lucas Moreau
PowerScore Staff
  • PowerScore Staff
  • Posts: 216
  • Joined: Dec 13, 2012
|
#17219
Hello, sotor26,

I'm glad you chose this question - I've taught this question in classes many times and know it pretty well by now! :-D

For Question 16: It is not necessarily true that only a linguistic monograph can be on the first shelf. The first shelf could also contain the Vonnegut novel. Since no two of V, P, and S can be on the same shelf as each other (combination of the second and third rules), one of each of them must be on each shelf - which means the first shelf must be P, S, or V. Does that make sense? :)



Hope that helps,
Lucas Moreau
 sotor26
  • Posts: 13
  • Joined: Oct 29, 2014
|
#17231
Thank you so much! It makes sense now! I can't believe I overlooked that.
 adlindsey
  • Posts: 90
  • Joined: Oct 02, 2016
|
#30949
Before I got to this question, I was looking at the set-ups for this game and I was wondering why W can't be on the 1st shelf?
 Emily Haney-Caron
PowerScore Staff
  • PowerScore Staff
  • Posts: 577
  • Joined: Jan 12, 2012
|
#31060
Hi adlindsey,

Great question! This is a tricky one, and requires an inference combining rules 2 and 3. We know that P and S can't be on the same shelf from rule 2, and we know that V can't be with P and V can't be with S from rule 3. The inference here is that V, P, and S all must be on different shelves, meaning that one of those three has to be on the first shelf. Since the first shelf has only one slot, that means no book BUT those three can be on the first shelf.

Hope that helps!
 adlindsey
  • Posts: 90
  • Joined: Oct 02, 2016
|
#31129
Thank you. I had a feeling it had to do with a combination of rules, but I couldn't see it.

Get the most out of your LSAT Prep Plus subscription.

Analyze and track your performance with our Testing and Analytics Package.